How can I show the equality,when a does not divide n?

  • MHB
  • Thread starter evinda
  • Start date
In summary: I used.What I did is a little more elementary, but it's really a question of what is taken as given.In summary, we have shown that:$$\left\lfloor \frac{\left\lfloor \frac{n}{a} \right\rfloor}{b} \right\rfloor = \left\lfloor \frac{n}{ab} \right\rfloor$$by showing that for any integer $q$ and any remainder $0 \leq r < a$, we have:$$q = \left\lfloor \frac{\left
  • #1
evinda
Gold Member
MHB
3,836
0
Hello! (Wave)

I want to show that:

$$\lfloor \frac{ \lfloor \frac{n}{a} \rfloor }{b} \rfloor = \lfloor \frac{n}{ab} \rfloor$$

That's what I have tried:

$$\lfloor \frac{n}{a} \rfloor =\max \{ m \in \mathbb{Z}: m \leq \frac{n}{a}\}$$

  • $a \mid n:$
    $$\lfloor \frac{n}{a} \rfloor=\frac{n}{a}$$

    Then:

    $$\lfloor \frac{ \lfloor \frac{n}{a} \rfloor }{b} \rfloor = \lfloor \frac{\frac{n}{a}}{b}\rfloor =\lfloor \frac{n}{ab} \rfloor $$
  • $a \nmid n:$
    $$\lfloor \frac{n}{a} \rfloor=\frac{n-k}{a},0<k\leq n$$
    $$\lfloor \frac{ \lfloor \frac{n}{a} \rfloor }{b} \rfloor= \lfloor \frac{n-k}{ab}\rfloor= \max \{ m \in \mathbb{Z}: m \leq \frac{n-k}{ab}\} (*) $$
But..to show what the exercise asks,I have to show that $(*)$ is equal to $\max \{ m \in \mathbb{Z}: m \leq \frac{n}{ab}\}$.

How can I do this? (Thinking)(Thinking)
 
Physics news on Phys.org
  • #2
evinda said:
  • $a \nmid n:$
    $$\lfloor \frac{n}{a} \rfloor=\frac{n-k}{a},0<k\leq n$$
    $$\lfloor \frac{ \lfloor \frac{n}{a} \rfloor }{b} \rfloor= \lfloor \frac{n-k}{ab}\rfloor= \max \{ m \in \mathbb{Z}: m \leq \frac{n-k}{ab}\} (*) $$
But..to show what the exercise asks,I have to show that $(*)$ is equal to $\max \{ m \in \mathbb{Z}: m \leq \frac{n}{ab}\}$.

How can I do this? (Thinking)(Thinking)

Hi! ;)

Suppose we pick $q$ and $r$ such that:
$$n=qab + r$$
with
$$0 \le r < ab$$

Then:
$$q=\left\lfloor \frac{n}{ab} \right\rfloor$$
and:
$$0 \le \frac ra < b$$

It follows that:
$$\left\lfloor \frac{ \left\lfloor \frac{n}{a} \right\rfloor}{b}\right\rfloor
= \left\lfloor \frac{ \left\lfloor \frac{qab + r}{a} \right\rfloor}{b}\right\rfloor
= \left\lfloor \frac{ qb + \left\lfloor \frac{r}{a} \right\rfloor}{b}\right\rfloor
= q + \left\lfloor \frac{ \left\lfloor \frac{r}{a} \right\rfloor}{b}\right\rfloor
= q
$$

In other words:
$$\left\lfloor \frac{n}{ab} \right\rfloor = \left\lfloor \frac{ \left\lfloor \frac{n}{a} \right\rfloor}{b}\right\rfloor$$
(Mmm)
 
  • #3
Just a thought:

If $n = qa + r$ with $0 \leq r < a$

then:

$\lfloor \frac{n}{a}\rfloor = \lfloor q + \frac{r}{a}\rfloor = q$

since $0 \leq \dfrac{r}{a} < 1$

Repeating, we have:

$q = q'b + r'$, so that:

$\lfloor \frac{q}{b}\rfloor = \lfloor q' + \frac{r,}{b}\rfloor = q'$

Now $n = qa + r = (q'b + r')a + r = (q')(ab) + ar' + r$

So now all you have to do is show that:

$ar' + r < ab$.
 
  • #4
I like Serena said:
Hi! ;)

Suppose we pick $q$ and $r$ such that:
$$n=qab + r$$
with
$$0 \le r < ab$$

Then:
$$q=\left\lfloor \frac{n}{ab} \right\rfloor$$
and:
$$0 \le \frac ra < b$$

It follows that:
$$\left\lfloor \frac{ \left\lfloor \frac{n}{a} \right\rfloor}{b}\right\rfloor
= \left\lfloor \frac{ \left\lfloor \frac{qab + r}{a} \right\rfloor}{b}\right\rfloor
= \left\lfloor \frac{ qb + \left\lfloor \frac{r}{a} \right\rfloor}{b}\right\rfloor
= q + \left\lfloor \frac{ \left\lfloor \frac{r}{a} \right\rfloor}{b}\right\rfloor
= q
$$

In other words:
$$\left\lfloor \frac{n}{ab} \right\rfloor = \left\lfloor \frac{ \left\lfloor \frac{n}{a} \right\rfloor}{b}\right\rfloor$$
(Mmm)

How did you get from this: $\displaystyle{ \left\lfloor \frac{ \left\lfloor \frac{qab + r}{a} \right\rfloor}{b}\right\rfloor }$ to this one: $ \displaystyle{ \left\lfloor \frac{ qb + \left\lfloor \frac{r}{a} \right\rfloor}{b}\right\rfloor } $?
 
  • #5
evinda said:
How did you get from this: $\displaystyle{ \left\lfloor \frac{ \left\lfloor \frac{qab + r}{a} \right\rfloor}{b}\right\rfloor }$ to this one: $ \displaystyle{ \left\lfloor \frac{ qb + \left\lfloor \frac{r}{a} \right\rfloor}{b}\right\rfloor } $?

We have:
$$
\left\lfloor \frac{qab + r}{a} \right\rfloor
=\left\lfloor qb + \frac{r}{a} \right\rfloor
$$
Since $qb$ is an integer, it does not affect the rounding, so we can take it out of the floor operation.
So:
$$\left\lfloor qb + \frac{r}{a} \right\rfloor
=qb + \left\lfloor \frac{r}{a} \right\rfloor
$$
 
  • #6
Deveno said:
Just a thought:

If $n = qa + r$ with $0 \leq r < a$

then:

$\lfloor \frac{n}{a}\rfloor = \lfloor q + \frac{r}{a}\rfloor = q$

since $0 \leq \dfrac{r}{a} < 1$

Repeating, we have:

$q = q'b + r'$, so that:

$\lfloor \frac{q}{b}\rfloor = \lfloor q' + \frac{r,}{b}\rfloor = q'$

Now $n = qa + r = (q'b + r')a + r = (q')(ab) + ar' + r$

I understood so far..

Deveno said:
So now all you have to do is show that:

$ar' + r < ab$.

But..why do I have to show now that $ar' + r < ab$ ? Could you explain it further to me? (Thinking)
 
  • #7
I like Serena said:
We have:
$$
\left\lfloor \frac{qab + r}{a} \right\rfloor
=\left\lfloor qb + \frac{r}{a} \right\rfloor
$$
Since $qb$ is an integer, it does not affect the rounding, so we can take it out of the floor operation.
So:
$$\left\lfloor qb + \frac{r}{a} \right\rfloor
=qb + \left\lfloor \frac{r}{a} \right\rfloor
$$

A ok! I understand.. (Happy) And also,why is it: $\displaystyle{ q+ \frac{ \lfloor \frac{r}{a} \rfloor }{b}}=q$ ? Has it to do with the fact that: $\displaystyle{ \frac{r}{a}<b}$ ? (Thinking)
 
  • #8
evinda said:
A ok! I understand.. (Happy) And also,why is it: $\displaystyle{ q+ \frac{ \lfloor \frac{r}{a} \rfloor }{b}}=q$ ? Has it to do with the fact that: $\displaystyle{ \frac{r}{a}<b}$ ? (Thinking)

That should be:
$${ q+ \left\lfloor\frac{ \lfloor \frac{r}{a} \rfloor }{b}\right\rfloor}=q$$
And yes, when we divide a numerator by a greater denominator and round down, the result is zero.
 
  • #9
I like Serena said:
We have:
$$
\left\lfloor \frac{qab + r}{a} \right\rfloor
=\left\lfloor qb + \frac{r}{a} \right\rfloor
$$
Since $qb$ is an integer, it does not affect the rounding, so we can take it out of the floor operation.

This is true, but depending on "what is taken as given" it may need justification.

In other words, there should be a lemma/theorem somewhere that:

If $k \in \Bbb Z$, then for any $x \in \Bbb R$, we have:

$\lfloor x+ k\rfloor = \lfloor x \rfloor + k$.

I presented what I did in two stages, for this reason-

At each stage, when I was evaluating an expression:

$\left\lfloor \dfrac{qa+r}{a} \right\rfloor$

I wanted the "remainder fraction" to be clearly between 0 and 1, so that the floor of:

$\dfrac{qa+r}{a}$ was clearly $q$.

Now, from what I posted before, we have:

$\left\lfloor \dfrac{\lfloor\frac{n}{a}\rfloor}{b} \right\rfloor = \lfloor \frac{q}{b}\rfloor = q'$

If we have that $0 \leq ar' + r < ab$, we have:

$0 \leq \dfrac{ar' + r}{ab} < 1$, which means that:

$\left\lfloor \dfrac{n}{ab} \right\rfloor = \left\lfloor q' + \dfrac{ar' + r}{ab}\right\rfloor = q'$, as well, so the two are equal.

It's not hard to show that $0 \leq ar' + r < b$:

We certainly have, from $r' < b$ that $r' \leq b - 1$, so that $ar' \leq a(b - 1) = ab - a$.

We also have $r < a$, so:

$ar' + r \leq ab - a + r < ab - a + a = ab$.

**********************

In all fairness, this is what ILS's equation:

$0 \leq \dfrac{r}{a} < b$ also accomplishes, for that means:

$0 \leq \dfrac{r}{ab} < \dfrac{b}{b} = 1$.
 
  • #10
I like Serena said:
That should be:
$${ q+ \left\lfloor\frac{ \lfloor \frac{r}{a} \rfloor }{b}\right\rfloor}=q$$
And yes, when we divide a numerator by a greater denominator and round down, the result is zero.

Oh yes,right! (Nod) So,can we say that $\frac{r}{a}<b \Rightarrow \lfloor \frac{r}{a} \rfloor <b$ ? (Thinking)

Also,doing it like that:

$$\lfloor \frac{ \lfloor \frac{n}{a} \rfloor}{b} \rfloor=\lfloor \frac{ \lfloor \frac{qab+r}{a} \rfloor}{b} \rfloor, \text{ with } 0 \leq r<ab$$
both of the cases $a \mid n$ and $a \nmid n$ are included,so we don't have to take cases,right? (Thinking)
 
  • #11
evinda said:
Oh yes,right! (Nod) So,can we say that $\frac{r}{a}<b \Rightarrow \lfloor \frac{r}{a} \rfloor <b$ ? (Thinking)

Yep. We can do it like that! (Happy)
More specifically:
$$\left\lfloor \frac{r}{a} \right\rfloor \le \frac{r}{a}$$
Also,doing it like that:

$$\lfloor \frac{ \lfloor \frac{n}{a} \rfloor}{b} \rfloor=\lfloor \frac{ \lfloor \frac{qab+r}{a} \rfloor}{b} \rfloor, \text{ with } 0 \leq r<ab$$
both of the cases $a \mid n$ and $a \nmid n$ are included,so we don't have to take cases,right? (Thinking)

Yep. We're covering both cases all at once. (Mmm)
 
  • #12
I like Serena said:
Yep. We can do it like that! (Happy)
More specifically:
$$\left\lfloor \frac{r}{a} \right\rfloor \le \frac{r}{a}$$

Yep. We're covering both cases all at once. (Mmm)

I understand! Thank you very much! (Cool)
 

FAQ: How can I show the equality,when a does not divide n?

1. How can I determine if a number is a multiple of another number?

To determine if a number a is a multiple of another number n, we can use the modulo operator (%). If the result of a % n is equal to 0, then a is a multiple of n.

2. What does it mean for a to not divide n?

If a does not divide n, it means that n cannot be divided evenly by a without leaving a remainder.

3. Can you provide an example of showing equality when a does not divide n?

Yes, for example, if a = 5 and n = 12, we can show equality by saying that 5 does not divide 12, since 12 divided by 5 leaves a remainder of 2.

4. How do I prove that a does not divide n?

To prove that a does not divide n, we can use the same method as in the first question. If a % n is not equal to 0, then a does not divide n.

5. Is it possible for a to not divide n and still have the two numbers be equal?

Yes, it is possible for a to not divide n and still have the two numbers be equal. This means that a and n have a common factor, but a is not a factor of n. For example, 6 and 15 are equal, but 3 does not divide 15 since it leaves a remainder of 3.

Similar threads

Replies
1
Views
1K
Replies
1
Views
1K
Replies
4
Views
1K
Replies
8
Views
1K
Replies
2
Views
2K
Replies
1
Views
1K
Replies
1
Views
938
Replies
1
Views
868
Replies
1
Views
1K
Back
Top